Induction Proof for Sequences Announcing the arrival of Valued Associate #679: Cesar Manara ...

Most bit efficient text communication method?

An adverb for when you're not exaggerating

Flight departed from the gate 5 min before scheduled departure time. Refund options

How many morphisms from 1 to 1+1 can there be?

What does Turing mean by this statement?

Tannaka duality for semisimple groups

Can the Flaming Sphere spell be rammed into multiple Tiny creatures that are in the same 5-foot square?

Is CEO the "profession" with the most psychopaths?

Time evolution of a Gaussian wave packet, why convert to k-space?

How can I set the aperture on my DSLR when it's attached to a telescope instead of a lens?

How does Belgium enforce obligatory attendance in elections?

Why are my pictures showing a dark band on one edge?

AppleTVs create a chatty alternate WiFi network

How does the math work when buying airline miles?

How to save space when writing equations with cases?

What's the difference between the capability remove_users and delete_users?

Dynamic filling of a region of a polar plot

Project Euler #1 in C++

What to do with repeated rejections for phd position

How to report t statistic from R

How were pictures turned from film to a big picture in a picture frame before digital scanning?

How does light 'choose' between wave and particle behaviour?

Misunderstanding of Sylow theory

Drawing spherical mirrors



Induction Proof for Sequences



Announcing the arrival of Valued Associate #679: Cesar Manara
Planned maintenance scheduled April 23, 2019 at 23:30 UTC (7:30pm US/Eastern)Find an explicit formula for the recursive sequenceGeneral formula for iterated cumulative sumUpper Bounds ProofShow That a Sequence is Monotonically IncreasingIs there a generalized analogue to the summation and product operators?How to prove this statement with “first” principle of Mathematical induction and not strong Mathematical induction?Sum of infinite series $sum_{i=1}^{infty} frac 1 {i(i+1)(i+2)…(i+n)}$Proof by induction of $s_k=2s_{k-2}$Sum of last digits of a sumProof by Induction: Recursively Defined Sequential Set












2












$begingroup$


Given a sequence $s_k=s_{k-1}+6k$, where $s_0=7$.



Question: First, find the closed formula for the $n$-th component of this sequence by hand and then prove that your formula is correct



My attempt:
I found the first couple of terms of the sequence to be
$s_0=7$,
$s_1=13$,
$s_2=25$,
$s_3=43$,
$s_4=67$ and
$s_5=97$.



I found the formula for the $n$-th term to be $s_n=3n^2+3n+7$.



Proof:
Base case $s_0=7$ therefore
$7=3cdot(0)^2+3cdot(0)+7$ so the formula works for the $s_0$ element.



I'm not sure how to proceed from here but I believe the proof should be a proof by Strong Induction. Any help will be greatly appreciated.










share|cite|improve this question









New contributor




Julia is a new contributor to this site. Take care in asking for clarification, commenting, and answering.
Check out our Code of Conduct.







$endgroup$

















    2












    $begingroup$


    Given a sequence $s_k=s_{k-1}+6k$, where $s_0=7$.



    Question: First, find the closed formula for the $n$-th component of this sequence by hand and then prove that your formula is correct



    My attempt:
    I found the first couple of terms of the sequence to be
    $s_0=7$,
    $s_1=13$,
    $s_2=25$,
    $s_3=43$,
    $s_4=67$ and
    $s_5=97$.



    I found the formula for the $n$-th term to be $s_n=3n^2+3n+7$.



    Proof:
    Base case $s_0=7$ therefore
    $7=3cdot(0)^2+3cdot(0)+7$ so the formula works for the $s_0$ element.



    I'm not sure how to proceed from here but I believe the proof should be a proof by Strong Induction. Any help will be greatly appreciated.










    share|cite|improve this question









    New contributor




    Julia is a new contributor to this site. Take care in asking for clarification, commenting, and answering.
    Check out our Code of Conduct.







    $endgroup$















      2












      2








      2


      1



      $begingroup$


      Given a sequence $s_k=s_{k-1}+6k$, where $s_0=7$.



      Question: First, find the closed formula for the $n$-th component of this sequence by hand and then prove that your formula is correct



      My attempt:
      I found the first couple of terms of the sequence to be
      $s_0=7$,
      $s_1=13$,
      $s_2=25$,
      $s_3=43$,
      $s_4=67$ and
      $s_5=97$.



      I found the formula for the $n$-th term to be $s_n=3n^2+3n+7$.



      Proof:
      Base case $s_0=7$ therefore
      $7=3cdot(0)^2+3cdot(0)+7$ so the formula works for the $s_0$ element.



      I'm not sure how to proceed from here but I believe the proof should be a proof by Strong Induction. Any help will be greatly appreciated.










      share|cite|improve this question









      New contributor




      Julia is a new contributor to this site. Take care in asking for clarification, commenting, and answering.
      Check out our Code of Conduct.







      $endgroup$




      Given a sequence $s_k=s_{k-1}+6k$, where $s_0=7$.



      Question: First, find the closed formula for the $n$-th component of this sequence by hand and then prove that your formula is correct



      My attempt:
      I found the first couple of terms of the sequence to be
      $s_0=7$,
      $s_1=13$,
      $s_2=25$,
      $s_3=43$,
      $s_4=67$ and
      $s_5=97$.



      I found the formula for the $n$-th term to be $s_n=3n^2+3n+7$.



      Proof:
      Base case $s_0=7$ therefore
      $7=3cdot(0)^2+3cdot(0)+7$ so the formula works for the $s_0$ element.



      I'm not sure how to proceed from here but I believe the proof should be a proof by Strong Induction. Any help will be greatly appreciated.







      sequences-and-series induction






      share|cite|improve this question









      New contributor




      Julia is a new contributor to this site. Take care in asking for clarification, commenting, and answering.
      Check out our Code of Conduct.











      share|cite|improve this question









      New contributor




      Julia is a new contributor to this site. Take care in asking for clarification, commenting, and answering.
      Check out our Code of Conduct.









      share|cite|improve this question




      share|cite|improve this question








      edited 11 hours ago









      Marian G.

      38426




      38426






      New contributor




      Julia is a new contributor to this site. Take care in asking for clarification, commenting, and answering.
      Check out our Code of Conduct.









      asked 11 hours ago









      JuliaJulia

      113




      113




      New contributor




      Julia is a new contributor to this site. Take care in asking for clarification, commenting, and answering.
      Check out our Code of Conduct.





      New contributor





      Julia is a new contributor to this site. Take care in asking for clarification, commenting, and answering.
      Check out our Code of Conduct.






      Julia is a new contributor to this site. Take care in asking for clarification, commenting, and answering.
      Check out our Code of Conduct.






















          3 Answers
          3






          active

          oldest

          votes


















          2












          $begingroup$

          You just apply the recurrence
          to your induction hypothesis.



          If
          $s_n=3n^2+3n+7
          $
          ,
          then,
          since
          $s_k=s_{k-1}+6k
          $
          ,



          $begin{array}\
          s_{n+1}
          &=s_n+6(n+1)\
          &=3n^2+3n+7+6(n+1)\
          &=3n^2+9n+13\
          end{array}
          $



          If your hypothesis is true,
          then



          $begin{array}\
          s_{n+1}
          &=3(n+1)^2+3(n+1)+7\
          &=3(n^2+2n+1)+3(n+1)+7\
          &=3n^2+6n+3+3n+3+7\
          &=3n^2+9n+13\
          end{array}
          $



          This matches the result
          from the induction step,
          so the induction hypothesis is proved.






          share|cite|improve this answer









          $endgroup$





















            2












            $begingroup$

            FYI, here is a way to determine what the closed formula is without having to determine it by hand. This is an extension of the answer given by szw1710. The given sequence is



            $$s_k=s_{k-1}+6k tag{1}label{eq1}$$



            Summing both sides of eqref{eq1} from $1$ to $n$ gives that



            $sum_{k=1}^{n}s_k = sum_{k=1}^{n}s_{k-1} + sum_{k=1}^{n}6k$



            $sum_{k=1}^{n-1}s_k + s_n = s_0 + sum_{k=1}^{n-1}s_{k} + 6frac{n(n+1)}{2}$



            $s_n = 7 + 3n(n+1) = 3n^2 + 3n + 7$



            As you can see, this technique can easily be used in any cases where you have $s_k = s_{k-1} + f(k)$ and the sum of $f(k)$ up to $k = n$ can be fairly easily determined.



            More generally, your question is a fairly simple example of Linear Recurrence Relations with Constant Coefficients. You can use a certain technique of a characteristic equation, as described in that link, to directly determine the solution of even considerably more complicated such equations.






            share|cite|improve this answer











            $endgroup$





















              2












              $begingroup$

              It seems to me that induction is not needed here. Fix $kinBbb N.$ A direct computation shows that $s_k-s_{k-1}=6k$ and $s_0=7.$ However, there is another problem: both induction, and my method show that if $s_k=3k^2+3k+7$, then $s_{k}=s_{k-1}+6k$. In fact, whe should prove the converse: if $s_{k}=s_{k-1}+6k$ and $s_0=7$, then $s_k=3k^2+3k+7$. I will look for some reasoning going in this direction.



              Let $s_0=7$ and $s_{k}=s_{k-1}+6k.$ Define $t_k=s_k-3k^2-3k-7.$ Then $t_0=0$. It is easy to prove (by direct computation) that $t_k=t_{k-1}$, so $(t_k)$ is a constant (in fact, zero) sequence. Then $s_k=3k^2+3k+7.$






              share|cite|improve this answer











              $endgroup$









              • 1




                $begingroup$
                My answer has one way to show what you're asking about, i.e., "if $s_{k}=s_{k-1}+6k$ and $s_0=7$, then $a_k=3k^2+3k+7$".
                $endgroup$
                – John Omielan
                9 hours ago










              • $begingroup$
                Yes, of course. About such techniques one could read in the "Concrete mathematics" book by Graham, Knuth, Patashnik.
                $endgroup$
                – szw1710
                9 hours ago












              Your Answer








              StackExchange.ready(function() {
              var channelOptions = {
              tags: "".split(" "),
              id: "69"
              };
              initTagRenderer("".split(" "), "".split(" "), channelOptions);

              StackExchange.using("externalEditor", function() {
              // Have to fire editor after snippets, if snippets enabled
              if (StackExchange.settings.snippets.snippetsEnabled) {
              StackExchange.using("snippets", function() {
              createEditor();
              });
              }
              else {
              createEditor();
              }
              });

              function createEditor() {
              StackExchange.prepareEditor({
              heartbeatType: 'answer',
              autoActivateHeartbeat: false,
              convertImagesToLinks: true,
              noModals: true,
              showLowRepImageUploadWarning: true,
              reputationToPostImages: 10,
              bindNavPrevention: true,
              postfix: "",
              imageUploader: {
              brandingHtml: "Powered by u003ca class="icon-imgur-white" href="https://imgur.com/"u003eu003c/au003e",
              contentPolicyHtml: "User contributions licensed under u003ca href="https://creativecommons.org/licenses/by-sa/3.0/"u003ecc by-sa 3.0 with attribution requiredu003c/au003e u003ca href="https://stackoverflow.com/legal/content-policy"u003e(content policy)u003c/au003e",
              allowUrls: true
              },
              noCode: true, onDemand: true,
              discardSelector: ".discard-answer"
              ,immediatelyShowMarkdownHelp:true
              });


              }
              });






              Julia is a new contributor. Be nice, and check out our Code of Conduct.










              draft saved

              draft discarded


















              StackExchange.ready(
              function () {
              StackExchange.openid.initPostLogin('.new-post-login', 'https%3a%2f%2fmath.stackexchange.com%2fquestions%2f3193879%2finduction-proof-for-sequences%23new-answer', 'question_page');
              }
              );

              Post as a guest















              Required, but never shown

























              3 Answers
              3






              active

              oldest

              votes








              3 Answers
              3






              active

              oldest

              votes









              active

              oldest

              votes






              active

              oldest

              votes









              2












              $begingroup$

              You just apply the recurrence
              to your induction hypothesis.



              If
              $s_n=3n^2+3n+7
              $
              ,
              then,
              since
              $s_k=s_{k-1}+6k
              $
              ,



              $begin{array}\
              s_{n+1}
              &=s_n+6(n+1)\
              &=3n^2+3n+7+6(n+1)\
              &=3n^2+9n+13\
              end{array}
              $



              If your hypothesis is true,
              then



              $begin{array}\
              s_{n+1}
              &=3(n+1)^2+3(n+1)+7\
              &=3(n^2+2n+1)+3(n+1)+7\
              &=3n^2+6n+3+3n+3+7\
              &=3n^2+9n+13\
              end{array}
              $



              This matches the result
              from the induction step,
              so the induction hypothesis is proved.






              share|cite|improve this answer









              $endgroup$


















                2












                $begingroup$

                You just apply the recurrence
                to your induction hypothesis.



                If
                $s_n=3n^2+3n+7
                $
                ,
                then,
                since
                $s_k=s_{k-1}+6k
                $
                ,



                $begin{array}\
                s_{n+1}
                &=s_n+6(n+1)\
                &=3n^2+3n+7+6(n+1)\
                &=3n^2+9n+13\
                end{array}
                $



                If your hypothesis is true,
                then



                $begin{array}\
                s_{n+1}
                &=3(n+1)^2+3(n+1)+7\
                &=3(n^2+2n+1)+3(n+1)+7\
                &=3n^2+6n+3+3n+3+7\
                &=3n^2+9n+13\
                end{array}
                $



                This matches the result
                from the induction step,
                so the induction hypothesis is proved.






                share|cite|improve this answer









                $endgroup$
















                  2












                  2








                  2





                  $begingroup$

                  You just apply the recurrence
                  to your induction hypothesis.



                  If
                  $s_n=3n^2+3n+7
                  $
                  ,
                  then,
                  since
                  $s_k=s_{k-1}+6k
                  $
                  ,



                  $begin{array}\
                  s_{n+1}
                  &=s_n+6(n+1)\
                  &=3n^2+3n+7+6(n+1)\
                  &=3n^2+9n+13\
                  end{array}
                  $



                  If your hypothesis is true,
                  then



                  $begin{array}\
                  s_{n+1}
                  &=3(n+1)^2+3(n+1)+7\
                  &=3(n^2+2n+1)+3(n+1)+7\
                  &=3n^2+6n+3+3n+3+7\
                  &=3n^2+9n+13\
                  end{array}
                  $



                  This matches the result
                  from the induction step,
                  so the induction hypothesis is proved.






                  share|cite|improve this answer









                  $endgroup$



                  You just apply the recurrence
                  to your induction hypothesis.



                  If
                  $s_n=3n^2+3n+7
                  $
                  ,
                  then,
                  since
                  $s_k=s_{k-1}+6k
                  $
                  ,



                  $begin{array}\
                  s_{n+1}
                  &=s_n+6(n+1)\
                  &=3n^2+3n+7+6(n+1)\
                  &=3n^2+9n+13\
                  end{array}
                  $



                  If your hypothesis is true,
                  then



                  $begin{array}\
                  s_{n+1}
                  &=3(n+1)^2+3(n+1)+7\
                  &=3(n^2+2n+1)+3(n+1)+7\
                  &=3n^2+6n+3+3n+3+7\
                  &=3n^2+9n+13\
                  end{array}
                  $



                  This matches the result
                  from the induction step,
                  so the induction hypothesis is proved.







                  share|cite|improve this answer












                  share|cite|improve this answer



                  share|cite|improve this answer










                  answered 11 hours ago









                  marty cohenmarty cohen

                  75.9k549130




                  75.9k549130























                      2












                      $begingroup$

                      FYI, here is a way to determine what the closed formula is without having to determine it by hand. This is an extension of the answer given by szw1710. The given sequence is



                      $$s_k=s_{k-1}+6k tag{1}label{eq1}$$



                      Summing both sides of eqref{eq1} from $1$ to $n$ gives that



                      $sum_{k=1}^{n}s_k = sum_{k=1}^{n}s_{k-1} + sum_{k=1}^{n}6k$



                      $sum_{k=1}^{n-1}s_k + s_n = s_0 + sum_{k=1}^{n-1}s_{k} + 6frac{n(n+1)}{2}$



                      $s_n = 7 + 3n(n+1) = 3n^2 + 3n + 7$



                      As you can see, this technique can easily be used in any cases where you have $s_k = s_{k-1} + f(k)$ and the sum of $f(k)$ up to $k = n$ can be fairly easily determined.



                      More generally, your question is a fairly simple example of Linear Recurrence Relations with Constant Coefficients. You can use a certain technique of a characteristic equation, as described in that link, to directly determine the solution of even considerably more complicated such equations.






                      share|cite|improve this answer











                      $endgroup$


















                        2












                        $begingroup$

                        FYI, here is a way to determine what the closed formula is without having to determine it by hand. This is an extension of the answer given by szw1710. The given sequence is



                        $$s_k=s_{k-1}+6k tag{1}label{eq1}$$



                        Summing both sides of eqref{eq1} from $1$ to $n$ gives that



                        $sum_{k=1}^{n}s_k = sum_{k=1}^{n}s_{k-1} + sum_{k=1}^{n}6k$



                        $sum_{k=1}^{n-1}s_k + s_n = s_0 + sum_{k=1}^{n-1}s_{k} + 6frac{n(n+1)}{2}$



                        $s_n = 7 + 3n(n+1) = 3n^2 + 3n + 7$



                        As you can see, this technique can easily be used in any cases where you have $s_k = s_{k-1} + f(k)$ and the sum of $f(k)$ up to $k = n$ can be fairly easily determined.



                        More generally, your question is a fairly simple example of Linear Recurrence Relations with Constant Coefficients. You can use a certain technique of a characteristic equation, as described in that link, to directly determine the solution of even considerably more complicated such equations.






                        share|cite|improve this answer











                        $endgroup$
















                          2












                          2








                          2





                          $begingroup$

                          FYI, here is a way to determine what the closed formula is without having to determine it by hand. This is an extension of the answer given by szw1710. The given sequence is



                          $$s_k=s_{k-1}+6k tag{1}label{eq1}$$



                          Summing both sides of eqref{eq1} from $1$ to $n$ gives that



                          $sum_{k=1}^{n}s_k = sum_{k=1}^{n}s_{k-1} + sum_{k=1}^{n}6k$



                          $sum_{k=1}^{n-1}s_k + s_n = s_0 + sum_{k=1}^{n-1}s_{k} + 6frac{n(n+1)}{2}$



                          $s_n = 7 + 3n(n+1) = 3n^2 + 3n + 7$



                          As you can see, this technique can easily be used in any cases where you have $s_k = s_{k-1} + f(k)$ and the sum of $f(k)$ up to $k = n$ can be fairly easily determined.



                          More generally, your question is a fairly simple example of Linear Recurrence Relations with Constant Coefficients. You can use a certain technique of a characteristic equation, as described in that link, to directly determine the solution of even considerably more complicated such equations.






                          share|cite|improve this answer











                          $endgroup$



                          FYI, here is a way to determine what the closed formula is without having to determine it by hand. This is an extension of the answer given by szw1710. The given sequence is



                          $$s_k=s_{k-1}+6k tag{1}label{eq1}$$



                          Summing both sides of eqref{eq1} from $1$ to $n$ gives that



                          $sum_{k=1}^{n}s_k = sum_{k=1}^{n}s_{k-1} + sum_{k=1}^{n}6k$



                          $sum_{k=1}^{n-1}s_k + s_n = s_0 + sum_{k=1}^{n-1}s_{k} + 6frac{n(n+1)}{2}$



                          $s_n = 7 + 3n(n+1) = 3n^2 + 3n + 7$



                          As you can see, this technique can easily be used in any cases where you have $s_k = s_{k-1} + f(k)$ and the sum of $f(k)$ up to $k = n$ can be fairly easily determined.



                          More generally, your question is a fairly simple example of Linear Recurrence Relations with Constant Coefficients. You can use a certain technique of a characteristic equation, as described in that link, to directly determine the solution of even considerably more complicated such equations.







                          share|cite|improve this answer














                          share|cite|improve this answer



                          share|cite|improve this answer








                          edited 9 hours ago

























                          answered 9 hours ago









                          John OmielanJohn Omielan

                          5,2442218




                          5,2442218























                              2












                              $begingroup$

                              It seems to me that induction is not needed here. Fix $kinBbb N.$ A direct computation shows that $s_k-s_{k-1}=6k$ and $s_0=7.$ However, there is another problem: both induction, and my method show that if $s_k=3k^2+3k+7$, then $s_{k}=s_{k-1}+6k$. In fact, whe should prove the converse: if $s_{k}=s_{k-1}+6k$ and $s_0=7$, then $s_k=3k^2+3k+7$. I will look for some reasoning going in this direction.



                              Let $s_0=7$ and $s_{k}=s_{k-1}+6k.$ Define $t_k=s_k-3k^2-3k-7.$ Then $t_0=0$. It is easy to prove (by direct computation) that $t_k=t_{k-1}$, so $(t_k)$ is a constant (in fact, zero) sequence. Then $s_k=3k^2+3k+7.$






                              share|cite|improve this answer











                              $endgroup$









                              • 1




                                $begingroup$
                                My answer has one way to show what you're asking about, i.e., "if $s_{k}=s_{k-1}+6k$ and $s_0=7$, then $a_k=3k^2+3k+7$".
                                $endgroup$
                                – John Omielan
                                9 hours ago










                              • $begingroup$
                                Yes, of course. About such techniques one could read in the "Concrete mathematics" book by Graham, Knuth, Patashnik.
                                $endgroup$
                                – szw1710
                                9 hours ago
















                              2












                              $begingroup$

                              It seems to me that induction is not needed here. Fix $kinBbb N.$ A direct computation shows that $s_k-s_{k-1}=6k$ and $s_0=7.$ However, there is another problem: both induction, and my method show that if $s_k=3k^2+3k+7$, then $s_{k}=s_{k-1}+6k$. In fact, whe should prove the converse: if $s_{k}=s_{k-1}+6k$ and $s_0=7$, then $s_k=3k^2+3k+7$. I will look for some reasoning going in this direction.



                              Let $s_0=7$ and $s_{k}=s_{k-1}+6k.$ Define $t_k=s_k-3k^2-3k-7.$ Then $t_0=0$. It is easy to prove (by direct computation) that $t_k=t_{k-1}$, so $(t_k)$ is a constant (in fact, zero) sequence. Then $s_k=3k^2+3k+7.$






                              share|cite|improve this answer











                              $endgroup$









                              • 1




                                $begingroup$
                                My answer has one way to show what you're asking about, i.e., "if $s_{k}=s_{k-1}+6k$ and $s_0=7$, then $a_k=3k^2+3k+7$".
                                $endgroup$
                                – John Omielan
                                9 hours ago










                              • $begingroup$
                                Yes, of course. About such techniques one could read in the "Concrete mathematics" book by Graham, Knuth, Patashnik.
                                $endgroup$
                                – szw1710
                                9 hours ago














                              2












                              2








                              2





                              $begingroup$

                              It seems to me that induction is not needed here. Fix $kinBbb N.$ A direct computation shows that $s_k-s_{k-1}=6k$ and $s_0=7.$ However, there is another problem: both induction, and my method show that if $s_k=3k^2+3k+7$, then $s_{k}=s_{k-1}+6k$. In fact, whe should prove the converse: if $s_{k}=s_{k-1}+6k$ and $s_0=7$, then $s_k=3k^2+3k+7$. I will look for some reasoning going in this direction.



                              Let $s_0=7$ and $s_{k}=s_{k-1}+6k.$ Define $t_k=s_k-3k^2-3k-7.$ Then $t_0=0$. It is easy to prove (by direct computation) that $t_k=t_{k-1}$, so $(t_k)$ is a constant (in fact, zero) sequence. Then $s_k=3k^2+3k+7.$






                              share|cite|improve this answer











                              $endgroup$



                              It seems to me that induction is not needed here. Fix $kinBbb N.$ A direct computation shows that $s_k-s_{k-1}=6k$ and $s_0=7.$ However, there is another problem: both induction, and my method show that if $s_k=3k^2+3k+7$, then $s_{k}=s_{k-1}+6k$. In fact, whe should prove the converse: if $s_{k}=s_{k-1}+6k$ and $s_0=7$, then $s_k=3k^2+3k+7$. I will look for some reasoning going in this direction.



                              Let $s_0=7$ and $s_{k}=s_{k-1}+6k.$ Define $t_k=s_k-3k^2-3k-7.$ Then $t_0=0$. It is easy to prove (by direct computation) that $t_k=t_{k-1}$, so $(t_k)$ is a constant (in fact, zero) sequence. Then $s_k=3k^2+3k+7.$







                              share|cite|improve this answer














                              share|cite|improve this answer



                              share|cite|improve this answer








                              edited 9 hours ago

























                              answered 11 hours ago









                              szw1710szw1710

                              6,5701223




                              6,5701223








                              • 1




                                $begingroup$
                                My answer has one way to show what you're asking about, i.e., "if $s_{k}=s_{k-1}+6k$ and $s_0=7$, then $a_k=3k^2+3k+7$".
                                $endgroup$
                                – John Omielan
                                9 hours ago










                              • $begingroup$
                                Yes, of course. About such techniques one could read in the "Concrete mathematics" book by Graham, Knuth, Patashnik.
                                $endgroup$
                                – szw1710
                                9 hours ago














                              • 1




                                $begingroup$
                                My answer has one way to show what you're asking about, i.e., "if $s_{k}=s_{k-1}+6k$ and $s_0=7$, then $a_k=3k^2+3k+7$".
                                $endgroup$
                                – John Omielan
                                9 hours ago










                              • $begingroup$
                                Yes, of course. About such techniques one could read in the "Concrete mathematics" book by Graham, Knuth, Patashnik.
                                $endgroup$
                                – szw1710
                                9 hours ago








                              1




                              1




                              $begingroup$
                              My answer has one way to show what you're asking about, i.e., "if $s_{k}=s_{k-1}+6k$ and $s_0=7$, then $a_k=3k^2+3k+7$".
                              $endgroup$
                              – John Omielan
                              9 hours ago




                              $begingroup$
                              My answer has one way to show what you're asking about, i.e., "if $s_{k}=s_{k-1}+6k$ and $s_0=7$, then $a_k=3k^2+3k+7$".
                              $endgroup$
                              – John Omielan
                              9 hours ago












                              $begingroup$
                              Yes, of course. About such techniques one could read in the "Concrete mathematics" book by Graham, Knuth, Patashnik.
                              $endgroup$
                              – szw1710
                              9 hours ago




                              $begingroup$
                              Yes, of course. About such techniques one could read in the "Concrete mathematics" book by Graham, Knuth, Patashnik.
                              $endgroup$
                              – szw1710
                              9 hours ago










                              Julia is a new contributor. Be nice, and check out our Code of Conduct.










                              draft saved

                              draft discarded


















                              Julia is a new contributor. Be nice, and check out our Code of Conduct.













                              Julia is a new contributor. Be nice, and check out our Code of Conduct.












                              Julia is a new contributor. Be nice, and check out our Code of Conduct.
















                              Thanks for contributing an answer to Mathematics Stack Exchange!


                              • Please be sure to answer the question. Provide details and share your research!

                              But avoid



                              • Asking for help, clarification, or responding to other answers.

                              • Making statements based on opinion; back them up with references or personal experience.


                              Use MathJax to format equations. MathJax reference.


                              To learn more, see our tips on writing great answers.




                              draft saved


                              draft discarded














                              StackExchange.ready(
                              function () {
                              StackExchange.openid.initPostLogin('.new-post-login', 'https%3a%2f%2fmath.stackexchange.com%2fquestions%2f3193879%2finduction-proof-for-sequences%23new-answer', 'question_page');
                              }
                              );

                              Post as a guest















                              Required, but never shown





















































                              Required, but never shown














                              Required, but never shown












                              Required, but never shown







                              Required, but never shown

































                              Required, but never shown














                              Required, but never shown












                              Required, but never shown







                              Required, but never shown







                              Popular posts from this blog

                              VNC viewer RFB protocol error: bad desktop size 0x0I Cannot Type the Key 'd' (lowercase) in VNC Viewer...

                              Tribunal Administrativo e Fiscal de Mirandela Referências Menu de...

                              looking for continuous Screen Capture for retroactivly reproducing errors, timeback machineRolling desktop...